American Math Competition (AMC) 10 A 2024 - Problem and Solution

Problem 1

What is the value of $9901 \cdot 101-99 \cdot 10101 ?$

(A) 2
(B) 20
(C) 200
(D) 202
(E) 2020

Problem 2

A model used to estimate the time it will take to hike to the top of the mountain on a trail is of the form $T=a L+b G$, where $a$ and $b$ are constants, $T$ is the time in minutes, $L$ is the length of the trail in miles, and $G$ is the altitude gain in feet. The model estimates that it will take 69 minutes to hike to the top if a trail is 1.5 miles long and ascends 800 feet, as well as if a trail is 1.2 miles long and ascends 1100 feet. How many minutes does the model estimates it will take to hike to the top if the trail is 4.2 miles long and ascends 4000 feet?

(A) 240
(B) 246
(C) 252
(D) 258
(E) 264

Problem 3

What is the sum of the digits of the smallest prime that can be written as a sum of 5 distinct primes?

(A) 5
(B) 7
(C) 9
(D) 10
(E) 13

Problem 4

The number 2024 is written as the sum of not necessarily distinct two-digit numbers. What is the least number of two-digit numbers needed to write this sum?

(A) 20
(B) 21
(C) 22
(D) 23
(E) 24

Problem 5

What is the least value of $n$ such that $n$ ! is a multiple of 2024 ?

(A) 11
(B) 21
(C) 22
(D) 23
(E) 253

Problem 6

What is the minimum number of successive swaps of adjacent letters in the string $A B C D E F$ that are needed to change the string to $F E D C B A$ ? (For example, 3 swaps are required to change $A B C$ to $C B A$; one such sequence of swaps is $
A B C \rightarrow B A C \rightarrow B C A \rightarrow C B A .)$

(A) 6
(B) 10
(C) 12
(D) 15
(E) 24

Problem 7

The product of three integers is 60. What is the least possible positive sum of the three integers?

(A) 2
(B) 3
(C) 5
(D) 6
(E) 13

Problem 8

Amy, Bomani, Charlie, and Daria work in a chocolate factory. On Monday Amy, Bomani, and Charlie started working at $1: 00 P M$ and were able to pack 4,3 , and 3 packages, respectively, every 3 minutes. At some later time, Daria joined the group, and Daria was able to pack 5 packages every 4 minutes. Together, they finished packing 450 packages at exactly $2: 45 P M$. At what time did Daria join the group?

(A) $1: 25 \mathrm{PM}$
(B) $1: 35 \mathrm{PM}$
(C) $1: 45 \mathrm{PM}$
(D) 1:55 PM
(E) 2:05 PM

Problem 9

In how many ways can 6 juniors and 6 seniors form 3 disjoint teams of 4 people so that each team has 2 juniors and 2 seniors?

(A) 720
(B) 1350
(C) 2700
(D) 3280
(E) 8100

Problem 10

Consider the following operation. Given a positive integer $n$, if $n$ is a multiple of 3 , then you replace $n$ by $\frac{n}{3}$. If $n$ is not a multiple of 3 , then you replace $n$ by $n+10$. Then continue this process. For example, beginning with $n=4$, this procedure gives $4 \rightarrow 14 \rightarrow 24 \rightarrow 8 \rightarrow 18 \rightarrow 6 \rightarrow$

$ 2 \rightarrow 12 \rightarrow \cdots$. Suppose you start with $n=100$. What value results if you perform this operation exactly 100 times?

(A) 10
(B) 20
(C) 30
(D) 40
(E) 50

Problem 11

How many ordered pairs of integers $(m, n)$ satisfy $\sqrt{n^2-49}=m$ ?

(A) 1
(B) 2
(C) 3
(D) 4
(E) Infinitely many

Problem 12

Zelda played the Adventures of Math game on August 1 and scored 1700 points. She continued to play daily over the next 5 days. The bar chart below shows the daily change in her score compared to the day before. (For example, Zelda's score on August 2 was $1700+80=1780$ points.) What was Zelda's average score in points over the 6 days?

(A) 1700
(B) 1702
(C) 1703
(D) 1713
(E) 1715

Problem 13

Two transformations are said to commute if applying the first followed by the second gives the same result as applying the second followed by the first. Consider these four transformations of the coordinate plane:

Of the 6 pairs of distinct transformations from this list, how many commute?

(A) 1
(B) 2
(C) 3
(D) 4
(E) 5

Problem 14

One side of an equilateral triangle of height 24 lies on line $\ell$. A circle of radius 12 is tangent to line $l$ and is externally tangent to the triangle. The area of the region exterior to the triangle and the circle and bounded by the triangle, the circle, and line $\ell$ can be written as $a \sqrt{b}-c \pi$, where $a$, $b$, and $c$ are positive integers and $b$ is not divisible by the square of any prime. What is $a+b+c$ ?

(A) 72
(B) 73
(C) 74
(D) 75
(E) 76

Problem 15

Let $M$ be the greatest integer such that both $M+1213$ and $M+3773$ are perfect squares. What is the units digit of $M$ ?

(A) 1
(B) 2
(C) 3
(D) 6
(E) 8

Problem 16

All of the rectangles in the figure below, which is drawn to scale, are similar to the enclosing rectangle. Each number represents the area of the rectangle. What is length $A B$ ?

Problem 17

Two teams are in a best-two-out-of-three playoff: the teams will play at most 3 games, and the winner of the playoff is the first team to win 2 games. The first game is played on Team A's home field, and the remaining games are played on Team B's home field. Team A has a $\frac{2}{3}$ chance of winning at home, and its probability of winning when playing away from home is $p$. Outcomes of the games are independent. The probability that Team A wins the playoff is $\frac{1}{2}$. Then $p$ can be written in the form $\frac{1}{2}(m-\sqrt{n})$, where $m$ and $n$ are positive integers. What is $m+n$ ?

(A) 10
(B) 11
(C) 12
(D) 13
(E) 14

Problem 18

There are exactly $K$ positive integers $b$ with $5 \leq b \leq 2024$ such that the base- $b$ integer $2024_b$ is divisible by 16 (where 16 is in base ten). What is the sum of the digits of $K$ ?

(A) 16
(B) 17
(C) 18
(D) 20
(E) 21

Problem 19

The first three terms of a geometric sequence are the integers $a, 720$, and $b$, where $a<720<b$. What is the sum of the digits of the least possible value of $b$ ?

(A) 9
(B) 12
(C) 16
(D) 18
(E) 21

Problem 20

Let $S$ be a subset of ${1,2,3, \ldots, 2024}$ such that the following two conditions hold:

What is the maximum possible number of elements in $S$ ?

(A) 436
(B) 506
(C) 608
(D) 654
(E) 675

Problem 21

The numbers, in order, of each row and the numbers, in order, of each column of a $5 \times 5$ array of integers form an arithmetic progression of length 5 . The numbers in positions $(5,5),(2,4),(4,3)$, and $(3,1)$ are $0,48,16$, and 12 , respectively. What number is in position $(1,2) ?$

(A) 19
(B) 24
(C) 29
(D) 34
(E) 39

Problem 22

Let $\mathcal{K}$ be the kite formed by joining two right triangles with legs 1 and $\sqrt{3}$ along a common hypotenuse. Eight copies of $\mathcal{K}$ are used to form the polygon shown below. What is the area of triangle $\triangle A B C$ ?

(A) $2+3 \sqrt{3}$
(B) $\frac{9}{2} \sqrt{3}$
(C) $\frac{10+8 \sqrt{3}}{3}$
(D) 8
(E) $5 \sqrt{3}$

Problem 23

Integers $a, b$, and $c$ satisfy $a b+c=100, b c+a=87$, and $c a+b=60$. What is $a b+b c+c a$ ?

(A) 212
(B) 247
(C) 258
(D) 276
(E) 284

Problem 24

A bee is moving in three-dimensional space. A fair six-sided die with faces labeled $A^{+}, A^{-}, B^{+}, B^{-}, C^{+}$, and $C^{-}$is rolled. Suppose the bee occupies the point $(a, b, c)$. If the die shows $A^{+}$, then the bee moves to the point $(a+1, b, c)$ and if the die shows $A^{-}$, then the bee moves to the point $(a-1, b, c)$. Analogous moves are made with the other four outcomes. Suppose the bee starts at the point $(0,0,0)$ and the die is rolled four times. What is the probability that the bee traverses four distinct edges of some unit cube?

(A) $\frac{1}{54}$
(B) $\frac{7}{54}$
(C) $\frac{1}{6}$
(D) $\frac{5}{18}$
(E) $\frac{2}{5}$

Problem 25

The figure below shows a dotted grid 8 cells wide and 3 cells tall consisting of $1^{\prime \prime} \times 1^{\prime \prime}$ squares. Carl places 1 -inch toothpicks along some of the sides of the squares to create a closed loop that does not intersect itself. The numbers in the cells indicate the number of sides of that square that are to be covered by toothpicks, and any number of toothpicks are allowed if no number is written. In how many ways can Carl place the toothpicks?

(A) 130
(B) 144
(C) 146
(D) 162
(E) 196

2022 AMC 10A, Problem 20, Hints and Solution

Motivation

To find the last term in a sequence, each term formed by adding similar indexed term from an AP and a GP.

Question

A four-term sequence is formed by adding each term of a four-term arithmetic sequence of positive integers to the corresponding term of a four-term geometric sequence of positive integers. The first three terms of the resulting four-term sequence are 57, 60 and 91. What is the fourth of this sequence?

Hint 1

Use the standard forms of the terms of the progressions to obtain a system of equations.

Hint 2

Try to reduce the number of variables from the system by subtracting two subsequent equations at a time. 

Let $a,ar,ar^2,ar^3$, be the first three terms of the geometric progression, and $b,b+d,b+2d,b+3d$ be the corresponding terms of the arithmetic progression.

We are given, that

$$a+b=57$$

$$ar+b+d=60$$

$$ar^2+b+2d=91.$$

Final Solution

These are 3 non-linear equations in 4 variables, so we can't directly conclude anything. Notice that if we subtract the first two equations we get, discarding $b$ $$3=a(r-1)+d$$ and similarly 

 $$31=ar(r-1)+d.$$

Each of these equations contain, the same variable. So subtracting again, we get

$$28=ar^2-2ar+a=a(r-1)^2.$$

Now since we're dealing with sequences of positive integers, then we can only equate $(r-1)^2$ to either $4$ or $1$.

Then we can conclude that either $a=28$ and $r=2$ or $a=7$ and $r=3$.

If $a=28$, then we get $b=57-28=29$ and $d=-25$. But that makes the arithmetic progression $29,4,-21,-46$, which is a contradiction since the sequence is of positive integers. With $a=7$, $b=50$, and $d=-11$ we get following progressions $50,39,28,17$ and $7,21,63,189$.

The desired number is then

$$17+189=206$$

AMC 10A 2002 Problem 15 | Prime Number

Try this beautiful Problem based on Number theory from AMC 10A, 2002 Problem 15.

Prime Number | AMC 10A 2021, Problem 15


Using the digits $1,2,3,4,5,6,7$, and 9 , form 4 two-digit prime numbers, using each digit only once. What is the sum of the 4 prime numbers?

Key Concepts


Arithmetic

Divisibility

Prime Number

Suggested Book | Source | Answer


Elementary Number Theory by David M. Burton.

AMC 10A 2002 Problem 15

190

Try with Hints


First try to find the probable digits for the unit place of the prime number.

The two digit prime number should end with $1, 3, 7, 9$ since it is prime and should not divisible by $2$ or $5$.

So now try to find which two digit primes will work here.

So, the primes should be $23, 41, 59, 67$.

Now find the sum of them.

AMC - AIME Program at Cheenta

Subscribe to Cheenta at Youtube


AMC 10A 2021 Problem 22 | System of Equations

Try this beautiful Problem based on System of Equations from AMC 10A, 2021 Problem 22.

System of Equations | AMC 10A 2021, Problem 22


Hiram's algebra notes are 50 pages long and are printed on 25 sheets of paper; the first sheet contains pages 1 and 2 , the second sheet contains pages 3 and 4 , and so on. One day he leaves his notes on the table before leaving for lunch, and his roommate decides to borrow some pages from the middle of the notes. When Hiram comes back, he discovers that his roommate has taken a consecutive set of sheets from the notes and that the average (mean) of the page numbers on all remaining sheets is exactly 19 . How many sheets were borrowed?

Key Concepts


Arithmetic Sequence

System of Equations

Algebra

Suggested Book | Source | Answer


Problem-Solving Strategies by Arthur Engel

AMC 10A 2021 Problem 22

13

Try with Hints


Let us assume that the roommate took sheets $a$ through $b$.
So, try to think what will be the changes in the page number?

So, page numbers $2 a-1$ through $2 b$. Because there are $(2 b-2 a+2)$ numbers.

Now apply the condition given there.

So we get, $\frac{(2 a-1+2 b)(2 b-2 a+2)}{2}$+$19(50-(2 b-2 a+2))$=$\frac{50 \cdot 51}{2}$

Now simplify this expression.

So , $2 a+2 b-39=25, b-a+1=13$

Now solve for $a, b$.

Find the number of pages using the values.

AMC - AIME Program at Cheenta

Subscribe to Cheenta at Youtube


AMC 10A 2021 I Problem 20 | Enumeration

Try this beautiful Problem based on Enumeration appeared in AMC 10A 2021, Problem 20.

AMC 10A 2021 I Problem 20


In how many ways can the sequence $1$, $2$, $3$, $4$, $5$ be rearranged so that no three consecutive terms are increasing and no three consecutive terms are decreasing?

Key Concepts


Permutation

Enumeration

Combinatorics

Suggested Book | Source | Answer


An Excursion in Mathematics

AMC 10A 2021 Problem 20

32

Try with Hints


We have 5 numbers with us.

So, how many permutations we can have with those numbers?

So, $5!=120$ numbers can be made out of those $5$ numbers.

Now we have to remember that we are restricted with the following condition -

no three consecutive terms are increasing and no three consecutive terms are decreasing.

Now make a list of the numbers which are satisfying the condition given among all $120$ numbers we can have.

Now the list should be -

$13254$, $14253$, $14352$, $15243$, $15342$, $21435$, $21534$, $23154$, $24153$, $24351$, $25143$, $25341$
$31425$, $31524$, $32415$, $32514$, $34152$, $34251$, $35142$, $35241$, $41325$, $41523$, $42315$, $42513$,
$43512$, $45132$, $45231$, $51324$, $51423$, $52314$, $52413$, $53412$.

Count how many permutations are there?

Subscribe to Cheenta at Youtube


AMC 10A 2021 Problem 14 | Vieta's Formula

Try this beautiful Problem based on Vieta's Formula from AMC 10A, 2021 Problem 14.

Vieta's Formula | AMC 10A 2021, Problem 14


All the roots of the polynomial $z^{6}$-$10 z^{5}$+$A z^{4}$+$B z^{3}$+$C z^{2}$+$D z+16$ are positive integers, possibly repeated. What is the value of $B$ ?

Key Concepts


Vieta's Formula

Polynomial

Roots of the polynomial

Suggested Book | Source | Answer


Problem-Solving Strategies by Arthur Engel

AMC 10A 2021 Problem 14

-88

Try with Hints


Find out the degree of the given polynomial.

We know, Degree of polynomial= Number of roots of that polynomial.

Apply Vieta's Formula on the given polynomial.

By Vieta's Formula, the sum of the roots is 10 and product of the roots is 16.

Since there are 6 roots for this polynomial. By trial and check method find the roots.

The roots should be $2, 2, 2, 2, 1, 1$.

Now using the roots reconstruct the polynomial.

So the polynomial should be -

$(z-1)^{2}(z-2)^{4}$

$=(z^{2}-2 z+1)\\(z^{4}-8 z^{3}+24 z^{2}-32 z+16)$

Now equate it with the given polynomial to find the value of $B.$

AMC - AIME Program at Cheenta

Subscribe to Cheenta at Youtube